XLRI Entrance Test XAT 2017 Data Interpretation and Quantitative Ability Question Paper With Answer Key

XLRI Entrance Test XAT 2017 Data Interpretation and Quantitative Ability
XLRI Entrance Test XAT 2017 Data Interpretation and Quantitative Ability Question Paper With Answer Key

XLRI Entrance Test XAT 2017

Data Interpretation and Quantitative Ability

1. The sum of series, (−100) + (−95) + (−95) + ….. + 110 + 115 + 120, is :

(A) 0

(B) 220

(C) 340

(D) 450

(E) None of the above

Answer: (C)

2. AB is a chord of a circle. The length of AB is 24 cm. P is the midpoint of AB. Perpendiculars from P on either side of the chord meets the circle at M and N respectively. If PM < PN and PM = 8 cm, then what will be the length of PN?

(A) 17 cm

(B) 18 cm

(C) 19 cm

(D) 20 cm

(E) 21 cm

Answer: (B)

3. If x and y are real numbers, the least possible value of the expression 4(x – 2)2 + 4(y – 3)2 – 2(x – 3)2 is :

(A) –8

(B) –4

(C) –2

(D) 0

(E) 2

Answer: (B)

4. Four two-way pipes A, B, C, and D can either fill an empty tank or drain the full tank in 4, 10, 12 and 20 minutes respectively. All four pipes were opened simultaneously when the tank is empty. Under which of the following conditions the tank would be half filled after 30 minutes?

(A) Pipe A filled and pipes B, C and D drained

(B) Pipe A drained and pipes B, C and D filled

(C) Pipes A and D drained and pipes B and C filled

(D) Pipes A and D filed and pipes B and C drained

(E) None of the above

Answer: (A)

5. A shop, which sold same market price shirts, announced on offer-if one buys three shirts then the fourth shirt is sold at a discounted price of Rs 100 only, Patel took the offer. He left the shop with 20 shirts after paying Rs 20,000. What is the marked price of a shirt ?

(A) Rs 1260

(B) Rs 1300

(C) Rs 1350

(D) Rs 1400

(E) Rs 1500

Answer: (B)

6. The volume of a pyramid with a square base is 200 cm3. The height of pyramid is 13 cm. What will be the length of the slant edges (i.e. the distance between the apex and any other vertex), rounded to the nearest integer?

(A) 12 cm

(B) 13 cm

(C) 14 cm

(D) 15 cm

(E) 16 cm

Answer: (C)

7. A dice is rolled twice. What is the probability that the number in the second roll will be higher than that in the first?

(A) 5/36

(B) 8/36

(C) 15/36

(D) 21/36

(E) None of the above

Answer: (C)

8. If f(x) = ax + b, a and b are positive real numbers and if

f(f(x)) = 9x + 8,

then the value of a + b is

(A) 3

(B) 4

(C) 5

(D) 6

(E) None of the above

Answer: (C)

9. Arup and Swarup leave point A at 8 AM to point B. To reach B, they have to walk the first 2 km, then travel 4 km by boat and complete the final 20 km by car. Arup and Swarup walk at a constant speed of 4 km/hr and 5 k m/hr respectively. Each rows his boat for 30 minutes. Arup drives his car at a constant speed of 50 km/hr while Swarup drives at 40 km/hr while Swarup drives at 40 km/hr. If no time is wasted in transit, when will they meet again

(A) At 9.15 AM

(B) At 9.18 AM

(C) At 9.21 AM

(D) At 9.24 AM

(E) At 9.30 AM

Answer: (D)

10. Hari’s family consisted of his younger brother (Chari), younger sister (Gouri) and their father and mother. When Chari was born, the sum of ages of Hari, his father and mother was 70 years. The sum of the ages of four family members, at the time of Gouri’s birth, was twice the sum of the ages of Hari’s father and mother at the time of Hari’s birth. If Chari is 4 years older than Gouri, then find the difference in age between Hari and Chari.

(A) 5 years

(B) 6 years

(C) 7 years

(D) 8 years

(E) 9 years

Answer: (E)

11. In a True/False quiz, 4 marks are awarded for each correct answer and 1 mark is deducted for each wrong answer. Amit, Benn and Chitra answered the same 10 questions, and their answer are given below in the same sequential order.

AMIT            T T F F T T F T T F

BENN           T T T F F T F T T F

CHITRA       T T T T F F T F T T

If Amit and Benn both score 35 marks each then Chitra’s score will be:

(A) 10

(B) 15

(C) 20

(D) 25

(E) None of the above

Answer: (A)

12. In a class of 60, along with English as a common subject, students can opt to major in Mathematics, Physics, Biology or a combination of any two. Six students major in both Mathematics and Physics, 15 major in both Physics and Biology, but no one majors in both Mathematics and Biology. In an English test, the average marks scored by students majoring in Mathematics is 45 and that of students majoring in Biology is 60. However, the combined average marks in English, of students of these two majors, is 50. What is the maximum possible number of students who major ONLY in Physics?

(A) 30

(B) 25

(C) 20

(D) 15

(E) None of the above

Answer: (D)

13. If 5° ≤ x° ≤ 15°, then the value of sin 30° + cos x° − sin x° will be :

(A) Between –1 and – 0.5 inclusive

(B) Between –0.5 and 0 inclusive

(C) Between 0 and 0.5 exclusive

(D) Between 0.5 and 1 inclusive

(E) None of the above

Answer: (E)

14. If N = (11p +7)(7q – 2 ) (5r + 1) (3s) is a perfect cube, where p, q, r and s are positive integers, then the smallest value of p + q + r + s is:

(A) 5

(B) 6

(C) 7

(D) 8

(E) 9

Answer: (E)

15. An institute has 5 departments and each department has 50 students. If students are picked up randomly from all 5 departments to form a committee, what should be the minimum number of students in the committee so that at least one department should have representation of minimum 5 students?

(A) 11

(B) 15

(C) 21

(D) 41

(E) None of the above

Answer: (C)

16. AB, CD and EF are three parallel lines, in that order. Let d1 and d2 be the distances from CD to AB and EF respectively. d1 and d2 are integers, where d1 : d2 = 2 : 1. P is a point on AB, Q and S are points on CD and R is a point on EF. If the area of the quadrilateral PQRS is 30 square units, what is the value of QR when value of SR is the least?

(A) slightly less than 10 units

(B) 10 units

(C) slightly greater than 10 units

(D) slightly less than 20 units

(E) slightly greater than 20 units

Answer: (E)

17. ABCD is a rectangle. P, Q and R are the midpoint of BC, CD and DA. The point S lies on e the line QR in such a way that SR : QS = 1 : 3. The ratio of the triangle APS and rectangle ABCD is

(A) 36/128

(B) 39/128

(C) 44/128

(D) 48/128

(E) 64/128

Answer: (A)

Directions (Q. 18 & 19) : Study the data given in the caselet below and answer these questions :

    In an innings of a T20 cricket match (a team an bowl for 20 overs) 6 bowlers bowled allowed maximum of 4 overs. Only the three specialist bowelers bowled their full quota of 4 overs each, and the remaining 8 overs were shared among three non-specialist bowlers. The economy rates of four bowlers were 6, 6, 7 and 9 respectively. (Economy rate is the total number of runs conceded by a bowler divided by the number of overs bowled by that bowler). This, however, does not include the data of the best bowler (lowest economy rate) and the worst bowler (highest economy rate). The number of overs bowled and the economy rate of any bowler are in integers.

18. Read the two statements below:

S1 : The worst bowler did not bowl the minimum number of overs.

S2 : The best bowler is a specialist bowler.

Which of the above statements or their combinations above statements or their combinations can help arrive at the minimum number of overs bowled by a non-specialist bowler?

(A) S1only.

(B) S2 only.

(C) .Either S1 or S2.

(D) S1 and S2 in combination

(E) The minimum number of overs can be determined without using S1 or S2.    

Answer: (E)

19. Read the two statements below:

S1. The economy rates of the specialist bowlers are lower than that of the non-specialist bowlers.

S2. The cumulative runs conceded by the three non-specialist bowlers were 1 more than those conceded by the three specialist bowlers.

Which of the above statements or their combinations can help arrive at the economy rate of the worst bowler?

(A) S1 only.

(B) S2 only.

(C) Either S1 or S2.

(D) S1 and S2 in combination.

(E) The economy rate can  be calculated without using S1 or S2.

Answer: (D)

Directions (Qs. 20 to 23) : Analyze the graph below and answer these questions :

  The grid below captures relationships among seven personality dimensions: “extraversion”, ‘true_arousal_plac”, “true_arousal_caff”, “arousal_plac”, “arousal_caff”, “performance_plac”, and “performance_caff”. The diagonal represents histograms of the seven dimensions. Left of the diagonal represents scatterplots between the dimensions while the right of the diagonal represents quantitative relationships between the dimensions. The lines in the scatterplots are closest approximation of the points. The value of the relationships to the right of the diagonal can vary from – 1 to  +1, with –1 being the extreme linear negative relation and +1 extreme linear positive relation. (Axes of the graph are conventionally drawn).

20. Which of the following is true?

(A) “Extraversion” has two modes.

(B) Median for “arousal_plac” is definitely the same as its average.

(C) Median for “arousal_caff” is definitely higher than its average.

(D) Median for “performance_plac” is definitely lower than its average.

(E) Median for “performance_caff’ is definitely lower than its average.

Answer: (D)

21. Which of the scatterplots shows the weakest relationships?

(A) Between “extraversion” and “peroformance_caff”.

(B) Between “true_arousal_plac” and “arousal_plac”.

(C) Between “true_arousal_plac” and “performance_plac”.

(D) Between “true_arosual_caff” and “performance_caff’.

(E) Between “arousal_caff” and “performance_caff”.

Answer: (A)

22. In which of the following scatterplots, the value of one dimension can be used to predict the value of another, as accurately as possible?

(A) “extraversion” and “true_arousal_caff”

(B) “true_arousal_plac” and “arousal_plac”

(C) “true_arousal_plac” and “performance_plac”

(D) “true_arousal_plac” and “performance_caff”

(E) All the above are irrelevant relations.

Answer: (C)

23. Which of the following options is correct?

(A) 0.93 on the right side of the scatterplot in the fourth row.

(B) 0.94 on the rightside of the diagonal corresponds to the second scatterponds to the second scatterplot in the fourth row.

(C) 0.38 is the relationship between “extraversion” and “true arousal_plac”.

(D) “arousal_caff” and “performance_caff” are positively related.

(E) The line that captures relationship between “arousal_caff” and “arousal_plac” can be denoted b y equation : y = a – bx, where b > 0.

Answer: (B)

Directions (Qs. 24 to 27) : Study the paragraph below and answer these questions :

Abdul has 8 factories, with different capacities, producing boutique kurtas. In the production process, he incurs raw material cost, selling cost (for packaging and transportation) and labour cost. These costs per kurta vary across factories. In all these factories, a worker takes 2 hours to produce a kurta. Profit per kurta is calculated by deducting raw material cost, selling cost and labour cost from the selling price (Profit = selling price – raw materials cost – selling cost – labour cost). Any other cost can be ignored.

24. Which of the following options is in decreasing order of raw materials cost?

(A) Factory 3, Factory, 4, Factory 7, Factory 5

(B) Factory 4, Factory 3, Factory 2, Factory 5

(C) Factory 6, Factory 3, Factory 5, Factory 7

(D) Factory 6, Factory 8, Factory 7, Factory 2

(E) Factory 8, Factory 3, Factory 2, Factory 4

Answer: (A)

25. Which of the factories listed in the options below has the lowest sales margin (sales margin = profit per kurta divided by selling price per kurta)?

(A) Factory 2

(B) Factory 4

(C) Factory 5

(D) Factory 6

(E) Factory 7

Answer: (E)

26. Abdul has received an order for 2,000 kurtas from a big retail chain. They will collect the finished pre-packaged kurtas directly from the factories, saving him the selling cost. To deliver this order, he can use multiple factories for production. Which of the following options will ensure maximum profit from this order?

(A) Factory 1

(B) Factories 2 and 3

(C) Factories 4 and 6

(D) Factories 3, 6 and 4

(E) Factory 1 or Factory 7 or Factory 8

Answer: (D)

27. Abdul has introduced a new technology in all his factories. As a result, a worker needs just 1.5 hours to produce a kurta. If raw materials cost and selling cost remain the same, which of the factories listed in the options below will yield the highest profit per kurta ?

(A) Factory 2

(B) Factory 3

(C) Factor 4

(D) Factory 5

(E) Factory 6

Answer: (B)

XLRI Entrance XAT 2017 Decision Making Ability Question Paper With Answer Key

XLRI Entrance XAT 2017 Decision Making Ability
XLRI Entrance XAT 2017 Decision Making Ability Question Paper With Answer Key

XAT (XLRI Entrance Test) 2017

DECISION MAKING ABILITY

Directions (1 & 2) : Analyse the following caselet and answer these questions:

    Vimla is the domestic help for Shreya and her neighbour Padma; both live in a posh gated community. Vimla not only cleans the house, but also cooks for both the families. Shreya treasures Vimla ever since she joined her family four years ago. Vimla joined Padm’s household this year.

1. One evening Shreya trying to pay the pizza delivery was surprised to find a few five hundred rupee notes missing from her purse that she was sure were there earlier. She wants to ascertain if someone has stolen the money. One reflecting, the following facts crossed her mind:

2. In the last one year, Shreya had noticed cash missing on three occasions.

3. Shreya’s husband also shared that a few notes were missing from his wallet, though he was not sure if they were stolen.

4. Her eldest son had been pestering Shreya for more pocket money for the last three weeks; in the last few days, he had stopped doing so.

5. In the last one year, Vimla had received six mails from her family asking for money.

1. Her eldest son’s expenditure had gone up in the last few days.

Which of the following combinations of the above statements would DECREASE the likelihood that Vimal has stolen the money ?

(A)  1 and 3

(B)  1 and 4

(C)  1 and 5

(D)  2 and 4

(E)  3 and 5

Answer: (E)

2. Padma discovered some money missing from her purse. She suspects that Vimla has stolen it. She wants to prevent the stealing from happening again and is contemplating the following actions:

1. She should let it pass, since to err is human.

2. She should confront Vimla and tell her that she knows the truth and the act is unpardonable regardless of her past service and she is thinking of terminating her services.

3. She should tell Vimla that she is aware someone has stolen money from the house but is not sure who it is

4. She should share with Vimla that neighbours think Vimla has stolen the money though she doesn’t, but is interested in finding out the truth.

5. She should directly ask Vimla if she stole the money, promising her no punishment if she confesses.

Arrange the following combinations of the above actions in the DECREASING order of appropriateness.

(A)  1, 5, 4

(B)  2, 4, 1

(C)  2, 5, 4

(D)  3, 5, 4

(E)  5, 2, 4

Answer: (D)

Analyse the following caselet and answer the question that follows:

3. Genius Consulting is a boutique consulting firm started by Shirish, Balram, Rahman and avier, four friends from a premier business school. They committed themselves to abide by two principles : a) not to indulge in anything unethical and b) share earnings equally.

     Genius Consulting could not get a significant project till the, following year, when they managed a big one after Rahman’s father referred their firm to his top management. Convinced of the team’s talent following an impressive presentation, the top management awarded them the project even though six other referred teams made presentations.

    The day following the presentation, they met to decide the way forward for the organization. Which of the following choices would be the most appropriate for Genius Consulting ?

(A)  As this project violates both their principles, Genius Consulting should not take up the project.

(B)  Due to the violation of the first principle Genius Consulting should not take up this project.

(C)  They should take up the project. Further, since Rahman had agreed to equal sharing, he is not entitled to finder’s fee.

(D)  They should take up the project and as the referral helped them survive, Rahman should be paid finder’s fee.

(E)  They should take up the project. But, in order not to violate the  principles, rahman can be paid finder’s fee this year and an equal amount be deducted, from h is compensation the next year.

Answer: (D)

Directions (Qs. 4 to 6) : Analyse the following caselet and answer these questions :

  Mrs. Biwas was to retire in one year after serving in the construction department of the Gujarat government for more than thirty years. After retirement, she wanted to spend her retired life along with Mr. Biswas, a retired school teacher in a small town in Kerala. They had two children, both studying in Bengaluru. The Biswas’ wished to construct a house in Kerala with their life savings.

1. Buy a fully furnished house from a big developer.

2. Buy a semi-furnished house from a big developer and furnish it.

3. Get a local unregistered contractor to construct a house and furnish it.

4. Mr. Biswas with inputs from the family could supervise the construction of a house back in Kerala by employing the best material, engineers, masons and labourers.

4. Which option would ENSURE the best control of quality of construction for the Biswas ?

(A)  The first option would ensure the best quality.

(B)  The second option would ensure the best quality.

(C)  The third option would ensure the best quality.

(D)  The fourth option would ensure the best quality.

(E)  Any of t he options will equally ensure the best quality.

Answer: (D)

5. Which of the following additional information, IF TRUE, would improve the chances of the third option being preferred ?

(A)  Based on the current information, with no additional information, the third is the  best option.

(B)  Among local property holders, the contractor in the third option enjoys a good reputation.

(C)  Big developers are less open to changes in design.

(D)  Mr. Biswas cannot stay back alone to supervise the construction.

(E)  The Biswas\ want to select the furniture on their own.

Answer: (B)

6. The Kerala Government recently announced a policy: In case of major quality infringement, the builder will pay a penalty of 50% of the price of the house in addition to the price of the house, to the client within a year of notice.

    Rank in ASCENDING order the options that would ensure “control of quality”.

(A)  1, 2, 3, 4

(B)  2, 3, 4, 1

(C)  4, 1, 2, 3

(D)  4, 3, 1, 2

(E)  4, 1, 2, 3

Answer: (C)

Directions (Qs. 7 & 8) : Analyse the following caselet and answer these questions :

    A pastor had eaten at a restaurant with his troupe of ten and his family. It is a norm to tip the waiter and about 20% of a waiter’s salary comes from these tips. However, while paying the bill, the pastor crossed out the automatic 18% tip charged for parties of more than eight and wrote “I give God 10% why do you get 18%?” above his signature. The chagrined waitress at the restaurant posted a photo of this on the social media. She was subsequently fired for violating company’s policy on customer privacy.

    This would have been understandable if the restaurant had not posted just 2 weeks ago a customer receipt that was complementing them. Social media and social activists came heavily upon the management’s ambivalent stand and the firing of the waitress. In response, the company posted a note on their social media page defending their actions. This quickly drew over 10,000 comments, mostly negative, to which the management started responding by posting the same note over and again. There were also accusations of the company deleting negative comments and blocking users. The restaurant also experienced a sizable drop in their football.

7. Who/what is the main cause for the situation becoming unmanageable ?

(A)  The pastor for flouting the norm of restaurant.

(B)  The waitress for violating customer privacy.

(C)  The management for not taking action against the pastor.

(D)  The management for giving out disproportionate punishment to waitress.

(E)  The management for removing negative comments from the social media.

Answer: (D)

8. The downward spiral continued for the restaurant as the management persisted in defending their actions and argued with those who criticized them. By the following week, the original post had generated over 18,000 negative comments.

    Which of the following is the best way forward for the restaurant at this juncture ?

(A)  Unconditionally restore the waitress to her former position and salary on the ground that she was never at fault in the first place.

(B)  Apologise to and reinstate the waitress and ask her to apologise for her breach of customer privacy and post both the apologies on social media.

(C)  Reinstate the waitress provided she apologises for her breach of customer privacy and post that apology on the restaurant’s social media page.

(D)  Reinstate the waitress if and only if she apologizes for her breach of customer privacy and posts that apology on her social media page.

(E)  Recruit two waitress at a higher wage but stick to the original decision of firing the waitress.

Answer: (B)

Directions (Qs. 9 & 10) : Analyse the following caselet and answer these questions :

  A college campus with a population of around 2,000 of whom 200 were children, 1,200 people between 15 and 45 years, 500 people between 45 and 65 and around 100 people more than 65 years of age. The campus has two big gates opening out to the city. There are 400 cars and 500 motorbikes inside the campus. The residents relied on these vehicles to visit the city located 10 kilometres away.

Now, with land within the campus becoming scarce, the chief administrator (CA) found the growing demand for parking lot difficult to handle. The faculty, staff and students wanted increased parking space. In the past six years, the parking requirement on campus had doubled. The CA found it inappropriate to construct parking lots from the students’ fee, even though those with vehicles may not complain about it.

   Besides creating parking problems, the CA felt that these vehicles added to the pollution and made residents less responsible towards each other. The risk to the children and the elderly because of over-speeding was menacing. Therefore, the CA wanted to reduce the number of vehicles in the campus. Many faculty members, students and staff members, however, felt that demand for more parking space was natural as vehicles were required to go to the market, railway station, airport, and inter-state bus terminal all located in the city. They also told the CA t hat the elderly, sick and toddlers relied only on these vehicles.

   After listening to all stakeholders, CA wanted to solve these problems while ensuring the campus remained responsible and green.

9. Which of the following actions would best satisfy all the stakeholders within the campus ?

(A)  Levy extra fee on students to construct additional parking lots inside the campus. Students should pay for up-keep of the campus.

(B)  Let all vehicles be parked in a college-managed space outside the campus while two battery operated vehicles handle exigencies inside the campus.

(C)  Bring all stakeholders to the discussion table.

(D)  To reduce carbon footprints, only pooled vehicles should be allowed to operate inside the campus.

(E)  Charge significant fee from any vehicle entering or leaving the campus.

Answer: (B)

10. Which of the following would be the best option to increase revenue, decrease carbon footprint and still satisfy all the stakeholders in the campus ?

(A)  Make substantial collection from vehicles entering or leaving campus and construct a parking lot outside the campus.

(B)  Levy extra fee on the students and build extra parking lots inside the campus.

(C)  Bring all stakeholders to table and let them build consensus.

(D)  To reduce the carbon footprint, only pooled vehicles be permitted on campus.

(E)  Let all vehicles be parked in a managed space outside the campus while battery operated vehicles parked at the two gates can be used inside the campus

Answer: (A)

Directions (Qs. 11 to 13) : Analyse the following caselet and answer these questions.

   Girirajan, an unemployed youngster from Kumbakonam, Tamilnadu, visited Singapore where his school friend worked as software engineer. In Singapore, Girirajan realized that there were a lot of Tamils in “Little India” area. He soon assessed that there were very few restaurants serving authentic Tamil food and decided to set up a restaurant “Giri’s” in “Little India” serving authentic Tamil food in traditional banana leaf. Customers loved the food. Very soon word spread about the good quality food served in the traditional way. Girirajan expanded operation recruiting 10 employees, all Tamils.

    Six months later, Girirajan realized that a lot of Kannadiga and Telugu customers started visiting the restaurant along with their Tamil friends.One day, a Kannadiga customer looking for Devangere Benne Dosa suggested that it may not be a bad idea for Girirajan so serve Karnataka and Andhra cuisines along with Tamil.

    With time, the popularity of the restaurant kept soaring. As a result, a lot of Malays, Indonesians and Chinese started visiting the restaurant. His software engineer friend advised to cater to customer of all nationalities. Despite his desire to go grand, Girirajan realized he did not have enough money to get extra space anywhere in Singapore and banks were also reluctant to l end.

   One day, while assessing the business, he realized that the restaurant had 90% occupancy rate during peak hours and 40% during the non-peak hours. Both figures were increasing with time.

11. Which of the following options would be most suitable for the growth of the business ?

(A)  Include Karnataka and Andhra cuisines in the menu.

(B)  Include Indonesian and Chinese cuisines in the menu.

(C)  Keep the focus on Tamil food only with an increased focus on quality improvement.

(D)  Expand to include either Karnataka and Andhra cuisines or Indonesian and Chinese cuisines.

(E)  Include Karnataka and Andhra cuisines as well as Indonesian and Chinese.

Answer: (C)

12. Girirajan started analyzing this quarterly customer data to asses growth opportunities. He discovered that 20% of his customers are Kannadigas and many of them were requesting for a few Karnataka dishes. he was not sure if it was a good idea to serve Karnataka food. he wanted to experiment before taking the final call. Which of the following options will be worth experimenting in the next fortnight ?

(A)  Introduce a Karnataka food day every week to please the Kannadiga customers.

(B)  Recruit a few Kannadiga waiters to interact with Kannadiga customers.

(C)  Rename a few Tamil dishes as Karnataka dishes.

(D)  Serve Karnataka dishes as and when requested by customers without the menu displaying these dishes.

(E)  Add a few Karnataka dishes to the cuisine and display these in a separate menu.

Answer: (E)

13. Girirajan experimented with Karnataka food for a fortnight. He collected sales data for Karnataka food along with the trend of overall sales for the last seven days. Which of t he following datasets, IF TRUE, will give Girirajan greater confidence to continue serving Karnataka food? (Figures below represent Karnataka food sales as percentage of total sales.)

(A)  14, 15, 16, 17, 18, 19, 20, with total sales trend increasing by 0.4%.

(B)  20, 20, 20, 20, 20, 20, 20, with total sales trend increasing by 0.4%.

(C)  50, 47, 44, 40, 47, 45, 40, with total sales trend remaining same.

(D)  45, 44, 43, 42, 41, 40, 39,, with total sales trend remaining same.

(E)  20, 50, 10, 45, 43, 13, 36, with total sales trend coming down.

Answer: (A)

Directions (Qs. 14 to 16) : Analyse the following caselet and answer these questions :

  A University coach was asked to select teams in three sports: Shooting, Cricket (batsmen only) and “Snakes & Ladder”. The  honest and keen observer head boy of the school informed the coach that he had observed 100 students playing the three games-Shooting, Cricket and “Snakes & Ladder”. In Shooting, all students were given 100 chances to hit a target. In Cricket, a batsman faced maximum of 100 balls, provided he DID NOT GET OUT. In “Snakes & Ladder”, every student could play 100 matches, one each with the other students and one against a computer. In Shooting, a player got one point for hitting the target and zero point for missing the target. In Cricket, a batsman got one point for hitting the ball and zero point for missing it. In “Snakes & Ladder”, a person got one point for winning the game and zero for losing. To the coach’s utter surprise, the distribution of points across all three games was the same. It was as follows :

The coach has to select a term of eleven in each sport.

14. Which of the following options is the best way to select the “Snakes & Ladder” team ?

(A)  The coach must select all students between 80 and 99.

(B)  The coach must select both the students between 90 and 99.

(C)  The coach must select at least 6 students between 40 and 59.

(D)  The coach must not select students between 0 and 9.

(E)  The coach can ignore the data in the table and randomly pick any 11 players.

Answer: (E)

15. Which of the following options is the best way of selecting the Shooting team?

(A)  The coach should select all students in 80-99 ranges.

(B)  The coach should select the both the students in 90-99 range; other can be from any of the other ranges.

(C)  The coach should select at least 6 students between 40 and 49 ranges.

(D)  The coach should not select students between 40 and 89 ranges.

(E)  The coach can pick randomly any 11 players from any of the ranges.

Answer: (E)

16. Which of the following options is the correct statements for picking up the cricket team?

(A)  The coach should never select the player in the range of 0-9 points.

(B)  The coach should never select players from the range of 50-59.

(C)  The coach should never select players from the range of 70-79.

(D)  The coach should definitely select players only from the range of 80-99.

(E)  None of the above

Answer: (E)

Analyse the following caselet and answer the question that follows:

17. As a newly started organization in the IT sector, Saksha IT is a fast growing 400 employees organization. Its head, Saksha Kumar believes in building an organization driven by humility. Which of the following policies would best help him achieve that?

(A)  Employees shall, in their annual evaluation form, be asked to acknowledge three of their shortcomings, overcoming which will make them better.

(B)  Employees shall be asked to share with their peers three shortcomings they had noticed about their peers.

(C)  The immediate superiors would suggest three shortcomings their subordinates have to work on in a given period.

(D)  Everyone beginning with Saksha shall be asked to share three of their weaknesses, in a public forum.

(E)  Employees shall be asked to overcome three shortcomings in the following year. Improvement, if noticed would attract rewards.

Answer: (D)

Analyse the following caselet and answer the question that follows :

18. Principal Maheswari of BINK School of Management is facing problem that calls for quick action. She received an anonymous mail one day, possibly from one of the students, complaining about outdated syllabi, mediocre teachers and poor choice of courses of offer.

     Which of the following is the best course of action available to the Principal?

(A)  She should invite students to discuss any problem openly, one on one or in small groups, promise to act on them but firmly condemn the culture of anonymous mails.

(B)  She should update the students about the efforts the institute is taking in sprucing up the syllabi, hiring talented faculty etc., but make no reference to the letter.

(C)  She should convene a meeting of the faculty at the earliest and get them to work on updating the syllabi and also arrange a faculty development program but keep mum about the letter.

(D)  She should convene a meeting of the faculty and talk about the letter as a symptom of a deeper malaise, ask them to update the syllabi and also organize a faculty development program.

(E)  She should ignore the letter and not think about it at all.

Answer: (A)

Directions (Qs. 19-21): Analyse the following caselet and answer these questions :

   Shyam prepares and sells piping hot puri and sabji (gravy) from 7:00 a.m. to 9:00 a.m. (both inclusive) through a stall at XLRI campus. Presently, he has just a single oven, which can do only one job at a time. It takes 30 minutes to cook one handi ( large bowl) of sabji, sufficient for 12 customers. It takes him 10 minutes to prepare 2 plates of puri at a time. While the cost of a plate of puri-sabji is Rs 10, the price is Rs 25. Puri stays hot for only 5 minutes after preparation, while sabji stays hot for 30 minutes only. It takes 10 minutes to reheat the sabji which stays hot for another 30 minutes. If he bbrings a handi of hot sabji from home, the transport would cost him Rs 50. It can also stay hot for 30 minutes excluding the transportation time. Currently, every 20 minutes five plates are ordered. All unsold quantities are distributed at cost after 9:30 a.m.

19. Find the maximum number of plates of puri-sabji Shyam can sell in the first hour of business?

(A)  6

(B)  8

(C)  10

(D)  12

(E)  15

Answer: (C)

20. Shyam’s puri-sabji became famous. Now he gets 5 orders per 10 minutes and has invested in an additional oven. As he is working alone on both the ovens simultaneously, now in any one oven it takes him 45 minutes to cook one handi of sabji, 15 minutes for 2 plates of puri and 15 minutes to reheat the sabji. He can bring only one handi from home. What will be his maximum daily profit?

(A)  Rs 250

(B)  Rs 280

(C)  Rs 300

(D)  Rs 330

(E)  Rs 360

Answer: (B)

21. Shyam’s stall was becoming popular by the day. Now he gets 5 customers every 10 minutes. After buying a second oven, he employed a helping hand, Aman at Rs 100 for two hours. Aman can cook one handi of sabji in 20 minutes or can prepare 3 plates of puri in 10 minutes. Reheat time for sabji is 10 minutes. None of them can work on both the ovens simultaneously.

    Which of the following options will help Shyam maximize his daily profit, if he does NOT bring sabji from  home?

(A)  Shyam should dismiss Aman because his wages are more than the benefits he brings.

(B)  Shyam will prepare only puri and Aman will prepare only sabji.

(C)  Shyam will prepare only sabji and Aman will prepare only puri.

(D)  Shyam and Aman will prepare both puri and sabji.

(E)  Options B, C and D will yield the same profit.

Answer: (D)

XLRI Entrance Jameshdpur, School of Management XAT 2017 Verbal and Logically Ability Question Paper and Answer Key

XLRI Entrance XAT 2017 Verbal and Logically Ability
XLRI Entrance Jameshdpur, School of Management XAT 2017 Verbal and Logically Ability Question Paper and Answer Key

XAT (XLRI Entrance Test) Examination-2017

XLRI Jamshedpur, School of Management

VERBAL AND LOGICAL ABILITY

1. Read the following statements and answer the question that follows:

1. This is Russia’s Wild West, though the mountains lie to the south of Moscow and St. Petersburg.

2. The Caucasus range has throughout history held Russians, especially fierce nationalists like Solzhenitsyn, in fear and awe.

3. Here, between the Black and Caspian Seas, is a land bridge where Europe gradually vanishes amid a six-hundred-mile chain of mountains as high as eighteen thousand feet-mesmerising in their spangled beauty, especially after the yawning and flat mileage of the steppe lands to the north.

4. Here, since the seventeenth century, Russian colonizers have tried to subdue congeries of proud peoples : Chechens, Ingush, Ossetes, Daghestanis, Abkhaz, Kartvelians, Kakhetians, Armenians, Azeris, and others.

5. Here the Russians encountered Islam in both its moderation and implacability.

Which of the following options is the best logical order of the above statements?

(A) 1, 2, 3, 4, 5

(B) 2, 3, 1, 4, 5

(C) 2, 4, 3, 1, 5

(D) 3, 1, 2, 4, 5

(E) 4, 5, 3, 1, 12

Answer: (B)

2. Read the following statements and answer the question that follows:

1. The periodic table orders the elements in a way that helps to understand why atoms behave as they do.

2. The properties of the elements are due to electronic configuration, and their recurring pattern gives rise to periodicity.

3. In other words, what gives the elements their properties and what order lies below the surface of their seemingly random nature?

4. What makes Fluorine react violently with Caesium while its nearest nieghbour neon is reluctant to react with anything?

Which of the following options is the best logical order of the above statements?

(A) 1, 2,3, 4

(B) 1, 4, 3, 2

(C) 2, 3, 1, 4

(D) 3, 4, 2, 1

(E) 4, 3, 1, 2

Answer: (C)

3. Read the following excerpt and answer t he question that follows:

Fragrant with steam were the days and the nights red in the beloved house of my father, my mother.

Which of the following options is the closest expression of the poet’s feeling?

(A) The house was located in beautiful settings probably surrounded by flowers in the mountains.

(B) The ancestral home was probably the most important hose in the community.

(C) The poet fondly recalls the pleasant climate enjoyed day and night.

(D) The poet misses the braziers and steam she had enjoyed during her childhood.

(E) Everything about the house felt special because of her parents’ love for her.

Answer: (D)

4. Read the following paragraphs and answer the question that follows:

    The current trend indicate that food and vegetable inflations continue to be pain points. Food inflation rose to 7.79 percent in June from 7.47 percent, and vegetable inflation rose to 14.74 percent from 10.85 percent. In the weeks ahead, the volatile food inflation will determine the course of overall inflation.

    For RBI too, the trend is a concern since under the current agreement with the government, if the inflation exceeds 11 percent it will have to explain to the government why it could not be contained (the lower limit is 2 percent)./

    Which of the following options is the most appropriate?

(A) The first paragraph states a cause and the second illustrates the effect.

(B) The first paragraph provides information and the second highlights potential application of the information.

(C) The first paragraph is an assertion and the second provides an illustration of that assertion.

(D) The first paragraph highlights inflation conditions and the second hints at RBI’s inefficiency in managing the situation.

(E) The first paragraph highlight pain points and the second contains a remedy for them.

Answer: (B)

5. The serious study of popular films by critics is regularly credited with having rendered obsolete a once-dominant view that popular mainstream films are inherently inferior to art films. Yet the change of attitude may be somewhat ______. Although, it is now academically respectable to analyse popular films, the fact that many critics feel compelled to rationalize their own _______ action movies or mass-market fiction reveals, perhaps unwittingly, their continued _______ the old hierarchy of popular and art films.

Consider the following words:

1. unproductive

2. not appreciated

3. overstated

4. penchant for

5. dislike for

6. investment in

7. exposure to

Which of the following options is the most appropriate sequence that would meaningful fit the blanks in the above paragraph?

(A) 1, 5, 6

(B) 3, 2, 7

(C) 3, 4, 6

(D) 4, 5, 6

(E) 6, 3, 1

Answer: (C)

6. Read the following stanza and answer the question that follows:

Invisible atoms coming together

Revealing themselves in visible forms

Seeds are hugged by the earth

Which renders them as gardens in bloom.

Any yonder stars, are they not pearls Floating on teeming seas?

Scattered, yet strung together in orderly constellations

Love binding them to one another

And each is perpetually seeking its like?

Which of the following options best captures the spirit of the above stanza?

(A) Stars and seas are similar.

(B) All rivers flow into the ocean.

(C) United we stand, divided we fall.

(D) Love dissolves all religious differences.

(E) Something invisible binds disparate objects.

Answer: (E)

7. Which of the following options is grammatically correct and meaningful?

(A) I want to joint an MBA college that is not only the best in the country but also provides the best campus jobs.

(B) I want to join an MBA college that is not only the best in the country but also I can get good job.

(C) I want to join an MBA college that is not only the best in the country but also best in job.

(D) I want to join an MBA college that is not only good but also I can get good job.

(E) I want to join an MBA college that I found not only best in country but also I can get good job.

Answer: (A)

8. Read the following passage and answer the question that follow:

On Friday morning, Dieting supplement sales company Herbalife agreed to pay the US Federal Trade Commission a $200m fine. The FTC said Herbalife cheated hopeful salespeople out of hundreds of millions of dollars with a high-pressure multi-level marketing scheme.

Herbalife’s stock received an immediate 15% increase following the above news. The company also announced that it would hire a second former FTC commissioner in a press release describing the terms of the settlement.

Which of the following options would imply that the 15 percent increase in stock  price is fair?

(A) Cheating results in increase in the company’s stock price.

(B) When fraudulent companies are exposed, their stock price increases.

(C) When fraudulent companies are caught, their stock price initially goes down.

(D) Acknowledgement of deceit increases the stock price of companies.

(E) Compliance with court order increases the company’s stock price.

Answer: (B)

9. Read the following paragraph and answer the question that follows :

Worldwide, tomato is one of the most important crops. Because this crop can be adapted for cultivation in various environment ranging from tropical to alpine regions, its cultivation area is now expanding worldwide into not so productive regions. On the other hand, traditional cultivation areas, the most favourable for tomato cultivation with warm and dry climate, are contracting. Every year, traditional cultivation areas lose 2 million hectares (ha) of land to environmental factors such as salinity, drought, and soil erosion.

Which of the following is the correct inference based on the above passage?

(A) In recent years, per hectare production of tomato has increased worldwide.

(B) In recent years, per hectare wastage of tomato has increased worldwide.

(C) In recent years, per hectare production of tomato has decreased worldwide.

(D) In recent years, per hectare wastage of tomato has decreased worldwide.

(E) In recent years, per hectare production of tomato has remained the same worldwide.

Answer: (C)

10. Carefully read the statements below:

1. Chatterjee loves books; therefore, he reads them all the time.

2. Chatterjee loves books. Therefore, he reads them all the time.

3. Chatterjee loves books and, therefore, reads them all the time.

Which of the above statement(s) is (are) correct in grammar and meaning?

(A) 1 only

(B) 2 only

(C) 1 and 2 only

(D) 2 and 3 only

(E) 1, 2 and 3

Answer: (D)

11. Grotesque is related to Macabre in a similar way as

(A) Classics is related to Ruins

(B) History is related to Palaeontology

(C) Marriage is related to Funeral

(D) Sorcery is related to Necromancy

(E) Science is related to Thanatology

Answer: (D)

12. Choose the option with all the (underlined syllable) that fits the blanks.

The suspension of the captain may _______ the number of spectators, who turn up for this match.

Transportation costs will directly _______ the cost of retail goods.

Grandmother’s advancing age could _____ her ability t take care of the house.

She _______ a Texan accent throughout the interview

(A) affect, effect, effect, effected

(B) affect, effect, affect, affected

(C) affect, affect, affect, affected

(D) effect, affect, effect, effected

(E) effect, affect, effect, affected

Answer: (C)

Directions (Qs. 13 to 15) : Analyse the passage below and answer these questions:

It’s taken me 60 years, but I had an epiphany recently. Everything, without exception, requires additional energy and order to maintain itself. I knew this in the abstract as the famous second law of thermodynamics, which states that everything is falling apart slowly. This realization is not just the lament of a person getting older. Long ago I learnt that even the most inanimate things we know of-stone, iron columns, copper pipes, gravel roads, a piece of paper-won’t last very long without attention and fixing and the loan of additional order. Existence, it seems, is chiefly maintenance.

     What was surprised me recently is how unstable even the intangible is. Keeping a website or a software program afloat is like keeping a yacht afloat. It is a black hole for attention. I can understand why a mechanical device like a pump would break down after a while-moisture rusts metal, or the air oxidizes membranes, or lubricants evaporate, all of which require repair. But I wasn’t thinking that the nonmaterial word of bits would also degrade. What’s to break? Apparently everything.

   Brand-new computers will ossify. Apps weaken with use. Code corrodes. Fresh software just released will immediately begin to fray. On their own-nothing you did. The more complex the gear, the more (not less) attention it will require. The natural inclination toward change is inescapable, even for the most abstract entities we know of: bits.

   And then there is the assault of the changing digital landscape. When everything around you is upgrading, this puts pressure on your digital system and necessitates maintenance. You may not want to upgrade, but you must because everyone else is. It’s an upgrade arms race.

    I used to upgrade my gear begrudgingly (why upgrade if it still works?) and at the last possible moment. You know how it goes: Upgrade this and suddenly you need to upgrade that, which triggers upgrades everywhere. I would put it off for years because I had  the experiences of one “tiny” upgrade of a minor part disrupting my entire working life. But as our personal technology is becoming more complex, more co-dependents upon peripherals, more like a living ecosystem, delaying upgrading is even more disruptive. If you neglect ongoing minor upgrades, the change backs up so much that the eventual big upgrade reaches traumatic  proportions. So I now see upgrading as type of hygiene: You do it regularly to keep your tech healthy. Continual upgrades are so critical for technological systems that they are now automatic for the major personal computer operating systems and some software apps. Behind the scenes, the machines will upgrade themselves, slowly changing their features over time. This happens gradually, so we don’t notice they are “becoming”.

     We take this evolution as normal.

       Technological life in the future will be a series of endless upgrades. And the rate of graduations is accelerating. Features shift, defaults disappear, menus morph. I’ll open up a software package I don’t use every day expecting certain choices, and whole menus will have disappeared.

      No matter how long you have been using a tool, endless upgrades make you into a newbie-the new user often seen as clueless. In this era of “becoming,” everyone  becomes a newbie. Worse, we will be newbies forever. That should keep us humble.

     That bears repeating. All of us-every one of us-will be endless newbies in the future in the future simply trying to keep up. Here’s why: First most of the important technologies that will dominate life 30 years from now have not yet been invented, so naturally you’ll be a newbie to them. Second, because the new technology requires endless upgrades, you will remain in the newbie state. Third, because the cycle of obsolescence is accelerating (the average lifespan of a phone app is a mere 30 days!), you won’t have time to master anything before it is displaced, so you will remain in the newbie mode forever. Endless Newbie is the new default for everyone, no matter your age or experience.

13. Which of the following statements would the author agree with the most?

(A) The second law of thermodynamics states that things need more energy as they separate.

(B) When it comes to erosion, intangibles behave differently from tangibles.

(C) Up-gradation is no longer an option but an obligation.

(D) Up-gradation though simple is disruptive.

(E) In the next thirty years, one’s experience in up-grading will be greatly valued.

Answer: (D)

14. Which of the following quotes would the author agree with the most?

(A) Life is like riding a bicycle. In order to avoid falling, you must keep moving.

(B) The only thing constant in life is change.

(C) You  must be the change you wish to see in the world.

(D) If you do not change you will be changed.

(E) What we can’t cure we must endure.

Answer: (B)

15. The CEO of a technology company was thinking of the following policies :

1. Life time employment

2. Promotion based on seniority

3. Hire new competent employees and fire old incompetent employees

4. Regular training and retraining

If a CEO were to consult the author of the passage, which of the above policies should the author recommend?

(A) 1 or 3

(B) 1 or 4

(C) 2 or 4

(D) 3 or 4

(E) 1, 3 and 4

Answer: (B)

Directions (Qs. 16 & 17) : Analyse the passage below and answer these questions:

   Writing is both my vocation and my avocation: that’s all I do.

    You may wonder why I should write a genealogy. Well, to begin with, my story is interesting. And, next, I am a mystery-more so than a tree or a sunset or even a flash of lightning. But, sadly, I am taken for granted by those who use me, as if I were a mere incident and without background. This supercilious attitude relegates me to the level of the commonplace. This is a species of the grievous error in which mankind cannot too long persist without peril. For, as a wise man, G. K. Chesterton, observed, “We are perishing for want of wonder, not for want of wonders.”

    I, simple though I appear to be, merit your wonder and awe, a claim I shall attempt to prove. In fact, if you can understand me-no, that’s too much to ask of anyone-if you can become aware of the miraculousness that I symbolize, you can help save the freedom mankind is so unhappily losing. I have a profound lesson than an automobile or an airplane or a mechanical dishwasher because- well, because I am seemingly to simple.

   Simple? Yet, not a single person on the face of this earth knows how to make me. This sounds fantastic, doesn’t it? Especially when you realize that there are about one-half billion of my kind produced in the US each year.

   Pick me up and look me over. What do you see? Not much meets the eye-there’s some wood, lacquer, the printed labeling, graphite lead, a bit of metal and an eraser.

16. “I” in the passage, most likely, refers to:

(A) the author of the passage

(B) a geometry box

(C) a study table

(D) a pencil

(E) the evolution of a book

Answer: (D)

17. A “supercilious attitude” in this passage implies:

(A) Failure to perceive the mystery of the sunset.

(B) Arrogance of treating all simple things as trivial.

(C) Lack of curiosity in seeking the mystery behind the lightning.

(D) A tendency to break down intricacies of creation into its simple  parts.

(E) Prosaic attitude immune to the mysteries of the world.

Answer: (B)

Directions (Q. 18 to 20) : Analyse passage below and answer these questions :

    Some psychologists and sociologists believe that psychopathy can be an asset in business and politics and that, as a result, psychopathic traits are over-represented among successful people. This would be a puzzle if it were so. If our moral feelings evolved through natural selection, then it shouldn’t be the case that one would flourish without them. And, in fact, the successful psychopath is probably the exception. Psychopaths have certain deficits. Some of these are subtle. The psychologist Abigail Marsh and her colleagues find that psychopaths are markedly insensitive to the expression of fear. Normal people recognize fear and treat it as a distress cue, but psychopaths have problems seeing it, let alone responding to it appropriately. Other deficits run deeper. The overall lack of moral sentiments-and specifically, the lack of regard for others-might turn out to be the psychopath’s downfall. We non-psychopaths are constantly assessing one another, looking for kindness and shame and the like, using this information to decide whom to trust, whom to affiliate with. The psychopath has to pretend to be one of us. But this is difficult. It’s hard to force yourself to comply with moral rules  just through a rational appreciation of what you are expected to do. If you feel like strangling the cat, it’s a struggle to hold back just because you know that it is frowned upon. Without a normal allotment of shame and guilt, psychopaths succumb to bad impulses, doing terrible things out of malice, greed, and simple boredom. And sooner or later, they get caught. While psychopaths can be successful in the short term, they tend to fail in the long term and often end up in prison or worse. Let’s take a closer look at what separates psychopaths from the rest of us. There are many symptoms of psychopathy, including pathological lying and lack of remorse or guilt, but the core deficit is indifference toward the suffering of other people. Psychopaths lack compassion. To understand how compassion works for all of us non-psychopaths, it’s important to distinguish it from empathy. Now, some contemporary researchers use the terms interchangeably, but there is a big difference between caring about a person (compassion) and putting yourself in the person’s shoes (empathy).

    I am too much of an adaptationist to think that a capacity as rich as empathy exists as a freak biological accident. It most likely has a function, and the most plausible candidate here is that it motivates us to care about others. Empathy exists to motivate compassion and altruism. Still, the link between empathy (in the sense of mirroring another’s feelings) and compassion (in the sense of felling and acting kindly toward another) is more nuanced than many people believe. First, although empathy can be automatic and unconscious-a crying person can affect your mood, even if you’re not aware that this is happening and would rather it didn’t-we often choose whether to empathize with another persons. So when empathy is present, it may be the product of a moral choice, not the cause of it. Empathy is also influenced by what one thinks of the other person. Second, empathy is not needed to motivate compassion. As the psychologist Steven Pinker points out, “If a child has been frightened by a barking dog and is howling in terror, my sympathetic response is not to howl in terror with her, but to comfort and protect her.” Third, just as you can have compassion without empathy, you can have empathy without compassion. You might feel the person’s pain and wish to stop feeling it-but choose to solve the problem by distancing yourself from that person instead of alleviating his or her suffering. Even otherwise good people sometimes turn away when faced with depictions of pain and suffering in faraway lands, or when passing a homeless person on a city street.

18. The core deficit of Psychopaths affects their long term success because,

(A) they cannot sustain the  behaviour.

(B) they are less likely to succeed as HR managers than as finance managers.

(C) the cannot hide their lack of compassion for long.

(D) empathy is essential for long term success.

(E) natural selection enables moral feelings.

Answer: (D)

19. Which of the following options is correct according to the author ?

(A) Compassion exists for a reason.

(B) Empathy is a chance event.

(C) Empathy is the cause of moral choice.

(D) Caring for others is psychopathy.

(E) Long term success in business is  a freak accident.

Answer: (C)

20. A student approached a faculty pleading to increase his marks because failure in one more subject will result in the student having to leave the program. The faculty said, “I am sorry. But I cannot change your grades as it would be unfair to others”.

In the given circumstance, which of the following best describes the faculty?

(A) The faculty is a psychopath.

(B) The faculty was compassionate.

(C) The faculty was both empathetic and compassionate but unfair.

(D) The faculty displayed empathy but not compassion.

(E) The faculty displayed compassion but not empathy.

Answer: (D)

Directions (Qs. 21 to 24) : Analyse the passage below and answer these questions :

    Every age has its pet contradictions. A few decades back, we used to accept Marx and Freud together, and then wonder, like the chameleon on the turkey carpet, why life was so confusing. Today there is similar trouble over the question whether there is, or is not, something called Human Nature. One the one hand, there has been an explosion of animal behaviour studies, and comparisons between animals and men have become immensely popular. People use evidence from animals to decide whether man is naturally aggressive, or naturally territorial; even whether he has an aggressive or territorial instinct. Moreover, we are still much influenced by Freudian psychology, which depends on the notion of instinct. On the other hand, many still hold what may be called the Blank Paper view, that man is a creature entirely without instincts. So do Existentialist philosophers. If man has no instincts, all comparison with animals must  be irrelevant. (Both these simple party lines have been somewhat eroded over time, but both are still extremely influential.)

     According to the Blank Paper view, man is entirely the product of his culture. He starts off infinitely plastic, and is formed completely  plastic, and is formed completely by the society in which he grows up. There is then no end to the possible variations among cultures; what we take to be human instincts are just the deep-dug prejudices of our own society. Forming families, fearing the dark, and jumping at the sight of a spider are just results of our conditioning. Existentialism at first appears a very different standpoint, because the Existentialist asserts man’s freedom and will not let him call himself a product of anything. But Existentialism too denies that man has a nature; if he had, his freedom would not be complete. Thus Sartre insisted that “there is no human nature……. Man first of all exists, encounters himself. surges up in the world, and defines himself afterwards. If man as the Existentialist sees him is not definable, it is because to begin with he is nothing. He will not be anything until later, and then he will be what he makes himself.” For Existentialism there is only the human condition, which is what happens to man and not what he is born like. If we are afraid of the dark, it is because we choose to be cowards; if we care more for our own children than for other people’s , it is because we choose to be partial ! We must never talk about  human nature or human instincts. This implicit moral notion is still very influential, not at all confined to those who use the metaphysic of essence and existence. So I shall sometimes speak of it, not as Existentialist, but as Libertarian-meaning that those holding it do not just (like all of us) think liberty important, but think it supremely important and believe that our having a nature would infringe it.

      Philosophers have not yet made much u se of informed comparison with other species as a help in the understanding of man. One reason they have not is undoubtedly the fear of fatalism. Another is the appalling way terms such as instinct and human nature have been misused in the past. A third is the absurdity of some ethological propaganda.

21. A business school led by a an existentialist director, wanted to decide on admission policy for its executive MBA program, which requires candidates to possess minimum five years of managerial experience.

   With respect to the selection process, which of the following statements will be closest to the director’s belief.

(A) Tenth standard marks should be given highest weightage.

(B) Twelfth standard marks engineering college should be given highest weightage.

(C) Marks scored in the engineering college should be given  highest weightage.

(D) Marks scored by the candidate in all previous examinations be given equal weightage.

(E) Recent work experience and contribution to the organization should be given highest weightage.

Answer: (E)

22. Which of the following statements would the author agree with the most ?

(A) Existentialism can be extended to Libertariansim.

(B) Existentialism and Libertarianism are the same.

(C) Existentialism encompasses Libertarianism.

(D) Animal behaviour should not be compared with human behaviour.

(E) Liberty and existentialism are unrelated.

Answer: (D)

23. Who among the following, as stated in the third paragraph, would the author be the most sympathetic to?

(A) PETA (People for the Ethical Treatment of Animals) activists

(B) Save the tiger activists

(C) Architect

(D) Physicists

(E) Zoologists

Answer: (A)

24. Which sentence in the passage distances man from “nature”, the most?

(A) The sixth sentence of the first paragraph.

(B) The first sentence of the second paragraph.

(C) The sentence third from the last in the passage.

(D) The sentence second from the last in the passage.

(E) The last sentence of the passage.

Answer: (D)

XLRI Entrance XAT 2016 Decision Making Question Paper and Answer Key

XAT 2016 - Decision Making
XLRI Entrance XAT 2016 Decision Making Question Paper and Answer Key

XAT (XLRI Entrance Test)

2016 Question Paper (Fully Solved)

XLRI Jamshedpur, School of Management

Decision Making

Directions (Qs. 1 & 2) : Analyze the following caselet and answer these questions.

     The City of Yashmund is served by licensed taxis operating on officially sanctioned metered rates and driven by licensed drivers who do not own the taxis but pay a monthly rent to the taxi-owners. Shaliesh Nair,, the mayor of Yashmund, perceived that most of these taxis do not offer sufficient comfort and safety to passengers.

1. The Mayor wants the owners and drivers to are about comfort.

Which of the following decisions, IF TAKEN, is MOST LIKELY to increase the comfort levels of passengers?

(A)  The mayor issues a guideline that taxies will be randomly inspected by the police for the comfort level.

(B)  The mayor ensures banks grant drivers loans to own cars. Owner-driven cars generally offer greater comfort.

(C)  The mayor introduces licensing of air-conditioned taxis which can charge increased rates to the rich customers.

(D)  The mayor introduces a feedback system that records passenger satisfaction with comfort levels; this will affect renewal of annual taxi licence.

(E)  The mayor permits doubling metered rates which will ensure enhanced income for owners to invest in greater comfort.

Ans: (D)

2. The mayor wants to involve the car owners in finding a solution to the problem of comfort and safety. he is concerned that the customers may not be willing to pay more for safety.

      Which of the options be low is MOST LIKELY to convince the owners?

(A)  The taxi owners who clear comfort-inspection can charge higher rentals from the drivers and drivers with impeccable safety record can charge the same from customers.

(B)  The taxis that clear comfort inspection can charge 25% above the metered rates; studies have shown that customers are willing to pay around 18% extra for comfort.

(C)  If a taxi owner has a consistent record of comfort and safety, the government will subsidise a second loan.

(D)  Taxis can charge 25% more if they clear comfort-inspection. However, owners of the taxis found compromising on safety will be jailed.

(E)  Taxis that pass comfort-inspection test can charge 25% more. Should they violate any traffic rule, this privilege would be withdrawn.

Ans: (D)

Directions (Qs. 3 & 4) : Analyze the following caselet and answer these questions:

     Chatterjee, the MLA of Trikathapur, owes his election success to his close friend and businessman Ghosh. The victory had appeared unlikely for Chatterjee after the arrival of Bhowmick, a budding politician with hordes of money. However, his clean image along with Ghosh’s money ensured Chatterjee’s resounding victory.

3. After the elections, Ghosh requested Chatterjee to sanction the land adjoining his factory, for expansion. However, the requested government land was a green belt reducing harmful pollution from the factory.

(A)  Chatterjee should approve the sale only after Ghosh plants a large number of frees around the factory and the city.

(B)  Chatterjee should oblige Ghosh provided he recruits 20 logcals as his employees on condition that they plant and maintain a tree each in their locality.

(C)  As Ghosh is paying market rates Chatterjee should approve the sale with no riders.

(D)  Chatterjee should approve the sale and ensure that the green belt is shifted to a different tract of land outside the city, purchased from the proceeds of the sale.

(E)  Chatterjee should unconditionally approve the transfer of the land to Ghosh as a token of gratitude.

Ans: (A)

4. Inspired by Bhowmick’s manifesto, Chatterjee is contemplating a green policy which can adversely affect Ghosh’s business interests. Which of the following actions form Ghosh is likely to convince Chatterjee NOT to pursue this policy?

(A)  Request Chatterjee to defer implementation of the green policy by  years, the time needed to make his factory green.

(B)  Remind Chatterjee that it is for his clean image that people voted him and not for Bhowmick’s green policy.

(C)  Warn Chatterjee that all industrialists will turn against him and despite his clean image he may be hated by the industry.

(D)  Appeal to Chatterjee’s sympathy citing the potential loss his business will suffer if the policy were to be implemented.

(E)  Threaten Chatterjee that he should not take his loyalty for granted as Bhowmick has invited him to join his party.

Ans: (B)

Directions (Qs. 5 to 7) : Analyze the following caselet and answer these questions.

   Indian Institute of Research is a Government-established body to promote research. In addition to helping in policy-making, it also provides free online access to all the articles to the public. It has a mission of publishing high-quality research articles. Till 2010, the publication of articles was very slow because there was no incentive for researchers to publish. Researchers stuck to the mandatory one article a year. Most of the researchers engaged in offering consultancy and earned extra income. Since its inception, the institute was considered the best place for cutting-edge research. The new director of the institute was not happy with the work done by researchers in silo and came out with a new research policy in 2013 to increase research output and improve collaboration among researchers. It was decided that extra benefits should be offered to researchers with new publications. As a result, the number of research articles increased fourfold in 2014. At the 2015 annual audit, an objection was raised against the new benefits scheme. Auditors were not happy with increased expenses towards remuneration for researchers. Further, the Government opined that the publication was itself a reward and hence researchers need be paid nothing extra. The director tried to defend his policy but the response from the Government was not encouraging.

Note : Auditor’s role is to verify accounts.

5. The following facts were observed by an analytics team hired by the government to study the extant situation.

1. There was a fourfold increase in the number of researchers leaving the organization in 2014.

2. A researcher died while on duty.

3. The quality of articles published declined substantially.

4. The average number of people accessing an article decreased by 2%.

Which of the following options would justify the government’s intention to DISCONTINUE the scheme?

(A)  1 and 2

(B)  2 and 3

(C)  3 only

(D)  4 only

(E)  3 and 4

Ans: (C)

6. The director still wanted to persuade the government to review its stand. he had framed the following arguments :

1. Most famous researchers in the world are also the highest paid.

2. American Institute of Research gives extra benefits to its scientists.

3. This year’s highest paid researcher had won the Noble Prize last year.

Considering the Government to be reasonable which of the following options is UNLIKELY to convince the Government?

(A)  1 and 2

(B)  2 only

(C)  2 and 3

(D)  1 and 3

(E)  1, 2 and 3

Ans: (E)

7. The director wanted to promote good decision-making at Indian Institute of Research. A few trusted colleagues offered the following suggestions:

1. Auditors need not be allowed to object to extra benefits schemes.

2. Auditors need not pin-point sudden increase in expenditure.

3. Auditors need not be consulted before taking any policy-level decision.

Which of the following combination of options should the director agree THE MOST with?

(A)  1 and 2

(B)  2 only

(C)  2 and 3

(D)  1 and 3

(E)  1, 2 and 3

Ans: (D)

Directions (Qs. 8 & 9) : Analyze the following caselet and answer these questions :

    Kamal Chinnappa, Vimal Rao, Ganesh Krishnan and Dinesh Kumar own a saloon each on the Barbil street. They are the only hairdressers on that street. Each of them offered three services viz. haircut, shaving and hair-dye. One evening, all four of them met in a nearby tea-stall and agreed to charge Rs. 100 for any of the three services (haircut, shave and hair-dye) on weekdays. They also agreed to increase this rate to Rs. 115 on weekends and holidays. All verbally decided to implement the agreement.

8. The following day Kamal, being the most competent hairdresser on the street, was contemplating charging higher than agreed upon price.

Which of the following would enable him to charge more with minimal violate of the agreement?

(A)  He should introduce a new and specialized service at Rs. 130.

(B)  He should open another shop on the same street and charge Rs. 150.

(C)  He should charge Rs. 130 for those wanting to jump the queue.

(D)  He should charge Rs. 115 for a service to a particular customer and give the next service free.

(E)  He should open his shop two hours before others and close it two hours after.

Ans: (A)

9. Vimal relies heavily on a bunch of loyal customers. He is concerned about retaining them.

Which of the following options should he choose if he does not want to violate the agreement?

(A)  He should charge differential rates for loyal customers

(B)  He should charge the loyal customers lower.

(C)  He should make every third visit free for this loyal customers.

(D)  He should charge all the agreed upon price.

(E)  He should allow his loyal customers to jump the queue.

Ans: (E)

Directions (Qs. 10 to 12) Analyze the following caselet and answer these questions:

Six people working at the Bengaluru office of Simsys are planning to buy flats at a real estate project at Whitefield. Their preferences are listed below:

Person Designation First Preference Second Preference Third Preference
Bhatia Vice President Ground floor flat Price < Rs. 50 lac. Shopping mall within 5 km.
Patel Client Relationship Manager Distance to office < 10 km Recreation Club
Khan Project Manager Recreation Club Place for morning walk Car parking
Singh Senior Software Engineer Shopping mall within 15 km Price < Rs. 30 lac Place for morning walk
Yadav Assistant software Engineer Price < Rs. 50 lac Distance to office < 10 km.
Lingdo Assistant Software Engineer Recreation Club

They have identified 7 real estate projects with following facilities available (marked with √):

Real Estate Project M N O P Q R S
Price Rs. 60-80 lac Rs. 45-50 lac Rs. 20-25 l;ac Rs. 65-80 lac Rs. 35-45 lac Rs. 25-40 lac Rs. 20-30 lac
Distance to office < 5 km < 10 km > 20 km > 15 km < 2 km < 10 km < 5 km
Place for morning walk
Recreation Club
Distance to shopping mall Inside > 25 km < 2 km Inside < 5 km > 10 km. > 20 km
Car parking facility
Availability of ground floor flat

A person ‘satisfied’ if a project meets all three preferences.

10. Identify the project(s) where NONE of the 6 persons will be ‘satisfied’.

(A)  M only

(B)  N only

(C)  P only

(D)  N and P only

(E)  In all projects at least one person will be ‘satisfied’.

Ans: (C)

11. Identify the project(s), where AT LEAST 3 of the 6 persons will be ‘satisfied’.

(A)  M only

(B)  S only

(C)  Q and R only

(D)  M, Q and S only

(E)  M, Q and R only

Ans: (D)

12. The marketing managers of all the six projects have agreed to add a recreation club and a car parking facility to the projects. In this changed scenario identify projects where AT MOST 2 of the 6 persons will NOT he ‘satisfied’?

(A)  N, Q and R only

(B)  P only

(C)  M and P only

(D)  N and P only

(E)  M, N and P only

Ans: (A)

Directions (Qs. 13 to 15) : Analyze the following caselet and answer these questions :

    Purushottam Bhatnagar owns and operates a sweet shop Puru and Sons. He is about 60 years old and is eager to hand over the business to his sons Ratan and Pramod. He, however, fears that his sons, fresh from college may not understand the tricks of the trade.

13. Purushottam sends a batch of sweets to the Police station across the street every day. Ratan construed it as bribe and wanted to stop this practice.

(A)  In the last three years, three attempts to burgle Puru and Sons were effectively foiled by the Police.

(B)  Each policeman receives only two pieces of sweet, too small to be considered a bribe.

(C)  The police in return send two policemen in mufti to mingle with the customers during rush hours to prevent pickpockets.

(D)  Every day, Purushottam also sends a batch of sweets to the school next to the station, an orphanage nearby and the temple at the end of the street.

(E)  Purushottam’s competitor Uttampurush who runs a sweetshop in the same street and his neighbour Mahapurush who runs a samosa stall, both do similar things every day.

Ans: (D)

14. Purushottam’s eldest son discovered that the shop repackaged sweets that were close to expiry and sold them at a discount under different names. These sweets usually get sold very fast. But his son was concerned about the possible consequences of this practice.

Purushottam was thinking of the following arguments to convince his son.

1. These sweets are consumed the same day and therefore there is no cause for worry.

2. Reduced prices give enough indication about the sweets to the customers.

3. These products are preferred by those who cannot afford full price and in a way, this is a service done to them.

4. In the past 30 years not a single person has reported ill because of consumption of these sweets.

5. Repackaging and selling sweets is a common practice.

Which combination of arguments below is MOST LIKELY to convince Ratan?

(A)  1 and 3

(B)  1 and 4

(C)  2 and 3

(D)  2 and 5

(E)  4 and 5

Ans: (B)

15. Purushottam’s younger son Pramod discovered that 10% of their customers who Purushotam called privileged customers purchased sweets at prices fixed 10 years ago (which is significantly lower than the current prices). Purushottam told him, “this 10% are my core and loyal customers with whom I have personal connect and therefore they deserve this privilege”, Pramod refuted his father’s argument citing the following information.

1. These customers form the top 20% of the income bracket of the city.

2. These customers frequently purchase from other sweetshops at market prices.

3. None of t hem recognizes and greets Purushottam at the shop or anywhere else.

4. None of the tem was present at Pramod’s marriage.

5. These customers actually buy sweets at Puru and Sons for others not part of the core and loyal customer group.

Which of the following combination of the above will MOST LIKELY convince Purushottam to charge market price to all?

(A)  1 and 2

(B)  2 and 4

(C)  2 and 5

(D)  3 and 4

(E)  4 and 5

Ans: (C)

Directions (Qs. 16 to 18) : Analyze the following caselet and answer these questions :

   Recently a private food testing agency reported the presence of a harmful chemical in Crunchy Chips, a product of a fast-moving consumer goods giant. The report sparked a nationwide outcry.

16. Rajan nShekawat, the CEO of the company, feared this incident might affect the company’s image among consumers. Rajan had the following options :

1. Apologising publicly for this inconvenience and immediately withdrawing the products from all stores.

2. Communicate the correct findings’ to the public.

3. Hire a reputed independent testing agency to verify the claims of the report.

4. Establish internal mechanisms to prevent repetition of such incidences in future.

5. Give higher incentives to distributors and retailers for selling the company brands.

  Which of the following would be the MOST APPROPRIATE ORDER of options for Rajan, starting from the immediate?

(A)  3, 1, 5

(B)  3, 2, 4

(C)  1, 3, 5

(D)  1, 2, 5

(E)  5, 3, 2

Ans: (B)

17. Mukesh Routray, a shopkeeper in a remote village was surprised to read in the newspaper, his only source of information, about harmful chemical in Crunchy Chips. He had stocked a large quantity of Crunchy Chips for the forthcoming festive season. He also r3ealised that people in his village are completely unaware of this controversy. He had the following options :

1. Sell the entire stock at a discount before the news spreads.

2. Destroy the entire stock and advise customers not to buy this product from other shops as well.

3. Donate the entire stock of Crunchy Chips to a local orphanage.

4. Inform customers about the controversy but understate it seriousness.

5. Ignore the news and sell the stock in the forthcoming festive season as planned.

6. Explore the veracity of the report and then take decision.

If arranged from ethical to unethical, which of the following is DEFINITELY the WRONG order?

(A)  6, 5, 1

(B)  6, 1, 4

(C)  4, 5, 1

(D)  2, 4, 3

(E)  2, 4, 1

Ans: (B)

18. An independent and trustworthy confidante of Rajan Shekhawat, the CEO of the company, informed him that one of their main competitors had bribed the food testing agency to manipulate the report.

Which of the following actions will BEST help Crunchy Chips to bounce back?

(A)  Proclaim over the media that their product is completely safe.

(B)  Secretly hire a food testing agency to ascertain the quality of the competitor’s product.

(C)  Hire another food testing agency to test and communicate the outcome to the consumers.

(D)  File a defamation case against the competitor for their alleged involvement in the conspiracy.

(E)  File a defamation case against the food testing agency.

Ans: (E)

Direction (Qs. 19 to 20)  : Analyze the following caselet and answer these questions :

 Nicky, Manoj and Benita are graduates from a top-ranked B-School. They joined ABC Corporation a year ago. ABC is known for its performance-oriented culture. This is the first time the organization recruited from a top-ranked B-School. They are part of a five-member team with two others from lower-ranked B-Schools. Nicky, Manoj and Benita draw 40 percent higher salaries than other team members. This team reports to Amelia Ganeshmurthi, a senior executive.

19. Amelia is disappointed with the performance of Nicky, Manoj and Benita. She came to know that ABC was not their first choice and they had spent the first ten months applying to other organizations. However, they have not started liking ABC and promised to do their best hence forth. Amelia has to rate their annual performance and decide about their future. She has the following choices:

1. Fir them from ABC for insincerity and save the organization’s time and money.

2. Give them average ratings with a year to prove their worth and fire them from ABC, if they fail to show significant progress.

3. Impose a pay-cut of 15% since they have not delivered on the promise, but give them relatively high ratings.

4. Give them relatively poor ratings with one-year time to improve and fire them from ABC if they fail to show significant progress.

5. Give them high ratings and give them a second chance to prove their worth.

Which of the following options rank the above choices in the order of MOST APPROPRIATE to LEAST APPROPRIATE?

(A)  1, 2, 4

(B)  2, 1, 4

(C)  4, 2, 3

(D)  4, 3, 1

(E)  5, 2, 3

Ans: (C)

20. Recruiting Nicky, Manoj and Benita was part of a larger initiative to make the organization attractive to prospective employees, Recently, Amelia’s boos informally told her that the trio’s perception of the organization might influence future recruitment from to B-Schools. However, the trio had already expressed their unhappiness about the organization to Amelia. She suspected that her promotion due next year might depend on the trio!

Which of the following is the BEST way for Amelia to deal with this situations?

(A)  Henceforth, she should be lenient with the trio.

(B)  She should promise the trio an early promotion if they can help her recruit good talent from top B-Schools.

(C)  Henceforth, she should occasionally invite the trio for dinner and informal outings.

(D)  She should tell her boss that it is unfair to link her promotion to the trio’s behaviour.

(E)  She should convey the trio’s unhappiness to her boss.

Ans: (E)

21. Nicky’s performance on the job is disappointing though she is considered a very helpful person outside the workplace helping her teammates and others in the organization with their personal needs, e.g. finding a place to rent, a good place to get homely food, etc. On the other hand, Manoj and Benita are performing well in their respective jobs and are perceived by their teammates as important to the team. But they are not interested in helping outside the workplace. Amelia has to decide the future of the trio. She has the following options :

1. Inform the higher authorities about Nicky’s poor performance and ask them to take a call.

2. Send Nicky for a one-month training earmarked for top-performing employees.

3. Serve Nicky an ultimatum to improve within the next six months or get fired.

4. Even though they performed well, give Manoj and Benita average ratings because of their disinterest in helping outside workplace.

5. Give Manoj and Benita high ratings based on their performance.

Which of the following combination of above options will be the MOST APPROPRIATE?

(A)  1 and 5

(B)  2 and 4

(C)  2 and 5

(D)  3 and 4

(E)  3 and 5

Ans: (E)

Directions (Qs. 22 & 23): Analyze the following caselet and answer these questions :

   Geetha Gawde can cultivate up to 6 crops a year. Crop A and B are ready for harvest in 2 moths; crop C and D in 3 months, and crop E and F in 4 months. Crop A can be cultivated from January to June; crop B can be cultivated from April to September, crop C can be cultivat3ed from May to December, crops D as well as E can be cultivated from August to December, and crop F from November to May. If Geetha plans a change of crop, the soil should be left fallow for one month; however, if the same crop is sown, no fallow time is needed. Sowing takes place only at the beginning of a month. Geetha can only harvest a maximum of 1000 units of any crop at any poit in time. The production cost per unit (incurred at the time of sowing) and price per unit  of crop are as follows :

Crop Production cost per unit crop (in USD) Price per unit crop (in USD)
A

B

C

D

D

F

20

5

25

15

5

35

60

55

70

75

65

15

For Geetha soil preparation does not incur any cost. If a crop is abandoned before the scheduled harvesting, she gets no money. Geetha is preparing a cropping schedule to maximize her annual profits (i.e. price-cost). She plans to replicate the schedule in the coming years.

22. Which of the following would DEFINITELY be a part of the ideal schedule?

(A)  Cultivate crop B in August or September.

(B)  Cultivate crop B from April to September.

(C)  Do not cultivate any crop in August but cultivate crop D in September

(D)  Cultivate crop D or crop E in August or September.

(E)  Do not cultivate any crop in August; but cultivate crop D or crop E in September.

Ans: (C)

23. Which of the following schedules would maximize her annual profit while minimizing the costs, if Geetha decides NOT to repeat a crop in a calendar year?

(A)  Crops A, B and E

(B)  Crops B, D and F

(C)  Crops B, D, E and F

(D)  Crops C, D and F

(E)  Crops A, B, D or E

Ans: (A)

XLRI Entrance XAT 2016 Verbal and Logically Ability Question Paper and Answer Key

XLRI Entrance XAT 2016
XLRI Entrance XAT 2016 Verbal and Logically Ability Question Paper and Answer Key

XAT (XLRI Entrance  Test)

2016 Question Paper (Fully Solved)

XLRI Jamshedpur, School of Management

Verbal and Logically Ability

1. “Assumptions are analogous to the basic ingredients in a gourmet recipe. Only the final product of the recipe dictat3es whether the ingredients suffice …..”

Which of the following is ANALOGOUS to the statement above?

(A)  Good wine needs no advertisement

(B)  The apple never falls far from the tree!

(C)  All is well that ends well!

(D)  As you sow, so shall you reap!

(E)  The proof of the pudding is in the eating!

Ans: (E)

2. Read the following poem and answer the question that follow :

I sought a soul in the sea

And found a coral there

Beneath the foam for me

An ocean was all laid bare,

 

Into my heart’s night

Along a narrow way

I groped; and lo! the light,

An infinite land of day.

Which of the following would best capture the ESSENCE of the poem above?

(A)  What lies ‘outside’ is always deceptive.

(B)  Pursue the narrow path and avoid the broadways.

(C)  External search is futile; explore the inner space for answers.

(D)  Heart’s pathways are broad and clear to find the destination.

(E)  Light offers sight and insight.

Ans: (C)

3. Which of the following options best captures the relationship similar to INSPECT : VIVISECT?

(A)  Enquire : Observe

(B)  Inquire : Explore

(C)  Investigate : Interrogate

(D)  Query : Survey

(E)  Question : Respond

Ans: (B)

4. The subject of this book is knavery, skullduggery, cheating, betrayal, unfairness, crime, sneakiness, malingering, cutting corner, immorality, dishonesty, betrayal, graft, wickedness, and sin.

Which of the following options best captures ALL the italicized words above?

(A)  Aggressive behaviours

(B)  Illegal behaviours

(C)  Deviant behaviours

(D)  Banned behaviours

(E)  Vetoed behaviours

Ans: (C)

5. Read the following conversation OINOS: I can comprehend you thus far – that certain operations of what we term Nature, or the natural laws, will under certain conditions, give rise to that which has all the appearance of creation. Shortly before the final overthrow of the earth, there were, I well remember many very successful experiments in what some philosophers were weak enough to denominate the creation of

AGATHOS: The cases of which  you speak were, in fact, instances of the secondary creation – and of the only species of creation which has ever been, since the first word spoke into existence the first law.

Which of the following options CANNOT be DEFINITELY inferred based on the above conversation?

(A)  Agathos was explaining something related to creation to Oinos.

(B)  At the time of conversation there was nothing called Earth.

(C)  The creation of animalculae is a natural law.

(D)  Natural laws are creations of philosophers.

(E)  Law is a spoken word.

Ans: (D)

6. Consider the two related statements below :

Statement I:

Offices and positions for the marginalized sections should be open to those with greater savings among them.

Statement II:

Offices and positions must be open to everyone based on the principle of fair opportunity.

(A)  Statement I assumes that the marginalized sections are incapable of saving.

(B)  Statement II assumes that all citizens are equally exposed to all opportunities

(C)  Statement II contradicts meritocracy.

(D)  Statement II assumes that all citizens are equally intelligent.

(E)  Statement I assumes that the marginalized sections always depend on subsidies.

Ans: (B)

7. This season will pass. The prime Minister may not win Lok Sabha elections, or she may; she may not continue as Prime Minister, or she may not continue as Prime Minister, or she may, The country will survive whatever the texture of politics in this decade or the next.

Which of the following, IF TRUE, will BEST reinforce the author’s view?

(A)  The survival of any Prime Minister is dependent on the country’s economic growth.

(B)  The country has a vibrant young working population.

(C)  The survival of the country depends on a dynamic, growth-oriented Prime Minister, not on the texture of politics.

(D)  The previous season had also witnessed similar political uncertainty.

(E)  The survival of the Prime Minister is dependent on the political texture of the country.

Ans: (D)

8. …there is a degree of convergence in the definition of trust which can be summarized as follows: Trust is a particular level of the subjective probability with which an agent assesses that another agent or group of agents will perform a particular action. When we say we trust someone or that someone is trustyworthy, we implicitly mean that the probability that he will perform an action that is beneficial to us..

Which of the following statements BEST COMPLETES the passage above?

(A)  is high enough for us to find out if he will cheat us.

(B)  is high enough for us to consider engaging in some form of cooperation with him.

(C)  is low enough for him not to engage in negative behavior against us.

(D)  is high enough for us not to build defences against his  possible aggression.

(E)  is low enough for us to attack him.

Ans: (B)

9. The FIRST and the LAST sentences of the paragraph are numbered 1 & 6. The others, labeled as P< Q, R and S are given below :

1. Suppose I know someone, Smith.

P. One day you come to me and say: “Smith is in Cambridge.”

Q. I inquire, and find  you stood at Guildhall and saw at the other end a man and said: “That was Smith”.

R. I’d say: “Listen. This isn’t sufficient evidence.”

S. I’ve heard that he has been killed in a battle in this war.

6. If we had a fair amount of evidence he was killed I would try to make you say that you’re being credulous.

Which of the following combinations is the MOST LOGICALLY ORDERED?

(A)  1SPQR6

(B)  1RSPQ6

(C)  1PRSQ6

(D)  1QSRP6

(E)  1RQPS6

Ans: (C)

10. The FIRST and the LAST sentences of the paragraph are numbered 1 & 6. The others, labeled as P, Q, R and S are given below:

1. The word “symmetry” is used here with a special meaning, and therefore needs to be defined.

P. For instance, if we look at a vase that is left and right symmetrical, then turn it 180° around the vertical axis, it looks the same.

Q. When we have a picture symmetrical, one side is somehow the same as the other side.

R. When is a thing symmetrical-how can we define it?

S. Professor Herman Weyl has given this definition of symmetry: a thing is symmetrical if one can subject it to a certain operation and it appears exactly the same after operation.

6. She shall adopt the definition of symmetry in Weyl’s more general form, and in that form we shall discuss symmetry of physical laws.

Which of the following combinations is the MOST LOGICALLY ORDERED?

(A)  1PQRS6

(B)  1QRRSP6

(C)  1RQPS6

(D)  1RQSP6

(E)  1SPQR6

Ans: (D)

11. In recent past, Indian football team has lost most of the matches in international football tournaments. The most successful coaches in Indian club football tournaments are form Latin American countries. In most of the Latin American countries, football is more popular sport than cricket.

From the passage above, choose the correct option :

(A)  It can be DEFINITELY concluded that “In India” cr3icket is more popular than football”

(B)  It can be DEFINITELY concluded that “Most Latin American countries are successful at football”.

(C)  It can be DEFINITELY concluded that “in recent past, coaches of Indian football teams are not from Latin America”.

(D)  It can be DEFINITELY concluded that “European football coaches are less successful than their Latin American counterparts for Indian national team.”

(E)  It cannot be DEFINITELY concluded that “The more popular a sport the better the chance of producing a successful coach in that sport.”

Ans: (E)

12. Choose the best pronunciation of the word, Sobriquet, from the following options:

(A)  soh-bri-key

(B)  sub-rry-ka

(C)  sob-bee-ri-kwet

(D)  soub-rick-kaat

(E)  sob-rik-kwet

Ans: (A)

Directions (Qs. 13 to 15) : Analyse the following passage and provide appropriate answers for these questions.

Advances in economic theory in the 1970s and 1980s illuminated the limits of markets; they showed that unfettered markets do not lead to economic efficiency whenever information is imperfect or markets are missing (for instance, good insurance markets to cover the key risks confronting individuals). And information is always imperfect and markets are always incomplete. Nor3 do markets, by themselves, necessarily lead to economic efficiency when the task of a country is to absorb new technology, to close the “knowledge gap”: a central feature of development. Today, most academic economists agree that markets, by themselves, do not lead to efficiency; the question is whether government can improve matters.

While it is difficult for economists to perform experiments to rest their theories, as a chemist or a physicist might, the world provides a vast array of natural experiments as dozens of countries try different strategies. Unfortunately, because each country differs in its history and circumstances and in the myriad of details in the policies-and details do matter-it is often difficult to get a clear interpretation. What is clear, however, is that there have been marked differences in performance, that the most successful countries have been those in Asia, and that in most of the Asian countries, government played a very active role. As we look more carefully at the effects of particular policies, these conclusions are reinforced: there is a remarkable congruence between what economic theory says government should do and what the East Asian governments actually did. By the same token, the economic theories based on imperfect information and incomplete risk markets that predicted that the free flow of shorter capital-a key feature of markets fundamentalist policies – would produce not growth but instability have also been borne out.

13. “… whether government can improve matters”. Here ‘matters’ indicates

(A)  Economic efficiency

(B)  Information imperfectness

(C)  Knowledge gaps

(D)  Good insurance markets

(E)  Incomplete risk markets

Ans: (A)

14. Which of the following options CANNOT be inferred from the above passage ?

(A)  Free flow of short-term capital might fail to ensure economic growth.

(B)  Insurance market is a proof that ‘markets, by themselves, do not lead to efficiency’.

(C)  It is difficult to interpret the success of economic policies of Asian countries.

(D)  Technology can impede market efficiency.

(E)  State intervention and imperfect information can never go hand-in-hand.

Ans: (E)

15. Which of the following statements BEST captures the ESSENCE of the two paragraphs in the above passage?

(A)  Paragraph I and Paragraph II are parallel arguments that are unrelated.

(B)  Paragraph I describes markets in general whereas Paragraph II describes market failures in Asian economics in particular.

(C)  Paragraph I explains why markets fail. Paragraph II spells out why market-based economic theories fail to explain success of Asian economics.

(D)  Paragraph I raises question and Paragraph II answers it.

(E)  Paragraph I states an economic theory and Paragraph II cites a natural experiment to disprove it.

Ans: (C)

Directions (Qs. 16 to 19) : Analyse the following passage and provide appropriate answers for these questions :

An effective way of describing what interpersonal communication is or is not, is perhaps to capture the underlying beliefs using specific game analogies.

       Communication as Bowling: The bowling model of message delivery is probably the most widely held view of communication. I think that’s unfortunate. This model sees the bowler as the sender, who delivers the ball, which is the message. As it rolls down the lane (the channel), clutter on the boards (noise) may deflect the ball (the message). Yet if it is aimed well, the ball strikes the passive pins (the target audience) with a predictable effect. In this one-way model of communication, the speaker (bowler) must take care to select a precisely crafted message (ball) and practice diligently to deliver it the same way every time,. Of course, that makes sense only if target listeners are interchangeable, static pins waiting to be bowled over by our words-which they aren’t.

      This has led some observes to propose an interactive model of interpersonal communication.

     Communication as Ping-Pong: Unlike bowling, Ping-Pong is not a solo game. This fact alone makes it a better analogy for interpersonal communication. One party puts the conversational b al in play, and the other gets into position to receive. It takes more concentration and skill to receive than to serve because while the speaker (server) knows where the message is going, the listener (receiver) doesn’t. Like a verbal or nonverbal message, the ball may appear straightforward yet have a deceptive spin.

     Ping-Pong is a back-and-forth game; players switch roles continuously. One moment the person holding the paddle is an initiator; the next second the same player is a responder, gauging the effectiveness of his or her shot by the way the ball comes back. The repeated adjustment essential for good play closely parallels the feedback process described in a number of interpersonal communication theories.

      Communication as Dumb Charades. The game of charades best captures the simultaneous and collaborative nature of interpersonal communication. A charade is neither in action, like bowling a strike, nor an interaction, lie a rally in Ping-Pong. It’s a transaction.

    Charades is a mutual game; the actual play is cooperative. One member draws a title or slogan from a batch of possibilities and then tries to act it out visually for teammates in a silent mini drama. The goal is to get at least one partner to say the exact words that are on the slip of paper. Of course, the actor is prohibited from talking out loud.

    Suppose you drew the saying “God helps those who help themselves.” For God you might try folding your hands and gazing upward. For helps you could act out offering a helping hand or giving a leg-up boost over a fence. By pointing at a number of real or imaginary people you may elicit a response of them, and by this point a partner may shout out, “God helps those who help themselves.” Success.

      Like charades, interpersonal communication is a mutual, on-going process of sending, receiving, and adapting verbal and nonverbal messages with another person to create and alter the images in both of our minds. Communication between us begins when there is some overlap between two images, and is effective to the extent that overlap increases. But even if our mental pictures are congruent, communication will be partial as long as we interpret them differently. The idea that “God helps those who help themselves” could strike one person as a hollow promise, while the other might regard it as a divine stamp of approval for hard work.

    Dumb Charade goes beyond the simplistic analogy of bowing and ping-pong. It views interpersonal communications as a complex transaction in which overlapping message simultaneously affect and are affected by the other person and multiple other factors.

16. The meaning CLOSEST to ‘interchangeable’ in the ‘Communication as Bowling’ paragraph is:

(A)  Complementary

(B)  Contiguous

(C)  Conforming

(D)  Compatible

(E)  Comparable

Ans: (E)

17. Which of the following options is the CLOSEST to the necessary condition of communication.

(A)  Threshold overlap of shared images

(B)  Simultaneous exchange

(C)  Ability to stimulate effect

(D)  Ability to enact a drama

(E)  Ability to elicit a response

Ans: (A)

18. The two inherent LIMITATIONS of Ping-Pong as a metaphor for communication are:

(A)  It is governed by conventions with possibility for appeal; it has clear rules.

(B)  The operating model in win-lose because only one individual or team can win; the receiver can always predict the spin.

(C)  The number of players is limited as very few can be meaningfully engaged at a time; the rules of the game are fixed by the regulators.

(D)  It demands more skills of the receiver than of the speaker; it is as passive as bowling.

(E)  Real life communications is like Dumb Charade wish multiple players; there are multiple balls used in Dumb Charade.

Ans: (C)

19. Action, interaction and transaction is CLOSEST to:

(A)  Advertising, Buyer negotiating with a seller, Bidding for a player in Indian Premier League.

(B)  Preparing an election manifesto, Addressing a public gathering, Engaging in door-to-door canvassing.

(C)  Preparing for MBA entrance exam, Writing the MBA entrance exam, Facing an interview for business school.

(D)  Applying for learner licence, Negotiating with a driving school, driving a car.

(E)  Negotiating overseas posting, Applying for visa, Undertaking a journey.

Ans: (A)

Directions (Qs. 20 to 22): Analyse the following passage and provide appropriate answers for these questions :

Each piece, or part, of the whole of nature is always merely an approximation to the complete truth, or the complete truth so far as we know it. In fact, everything we know is only some kind of approximation, because we know that we do not know all the laws as yet. Therefore, things must be learned only to be unlearned again or, more likely, to be corrected.

   The principle of science, the definition, almost, is the following: The test of all knowledge is experiment. Experiment is the sole judge of scientific “truth”. But what is the source of knowledge? Where do the laws that are to be tested come from? Experiment, itself, helps to produce these laws, in the sense that it gives us hints. But also needed is imagination to create from these hints the great generalizations- to guess at the wonderful, simple, but very strange patterns beneath them all, and then to experiment to check again whether we have made the right guess. This imagining process is so difficult that the4re is a division of labour in physics: there are theoretical physicists who imagine, deduce, and guess at new laws, but do not experiment; and then there are experimental physicists who experiment, imagine, deduce, and guess.

   We said that the laws of nature are approximate: that we first find the “wrong” ones, and then we find the “right” ones. Now, how can an experiment be “wrong”? First, in a trivial way: the apparatus can be faulty and you did no notice. But these things are easily fixed and checked back and forth. So without snatching at such minor things, how can the results of an experiment be wrong? Only by being inaccurate. For example, the mass of an object never seems to change; a spinning top has the same weight as a still one. So a “law” was invented: mass is constant, independent of speed. That “law” is now found to be incorrect. Mass is found to increase with velocity, but appreciable increase requires velocities near that of light. A true law is: if an object moves with a speed of less than one hundred miles a second the mass is constant to within one part in a million. In some such approximate form this is a correct law. So in practice one might think that the new law makes no significant difference. Well, yes and n. For ordinary speeds we can certainly forget it and use the simple constant mass law as a good approximation. But for high speeds we are wrong, and the higher the speed, the more wrong we are.

    Finally, and most interesting, philosophically we are completely wrong with the approximate law. Our entire picture of the world has to be altered even though the mass changes only by a little bit. This is a very peculiar thing about the philosophy, or the ides, behind the laws. Even a very small effect sometimes requires profound changes to our ideas.

20. Which of the following options is DEFINITELY NOT an approximation to the complete truth ?

(A)  I know that I know.

(B)  I know that I do not know.

(C)  I know what I know.

(D)  I know what I do not know.

(E)  I know that others do not know.

Ans: (D)

21. Consider the two statements from the passage:

Statement I:

The mass of an object never seems to change.

Statement II:

Mass is found to increase with velocity. Which of the following options CANNOT be concluded from the above passage?

(A)  Both statements I and II are approximation to the complete truth.

(B)  Both statements I and II are complete truth so far as we now.

(C)  Statement I is an approximation to the complete truth but Statement II is complete truth.

(D)  Statement I reveals that experimental physicists who imagine, deduce, and guess are philosophically wrong.

(E)  Statement II shows that theoretical physicists can pinpoint the shortcomings of experimental physicists.

Ans: (E)

22. ‘Big Bang’ is a popular theory related to the origin of the universe. It states that the universe was the outcome of a big bang that released enormous energy.

Which of the following is the MOST PROBABLE inference about the big bang theory?

(A)  Big Bang Theory was first proposed by experimental physicists.

(B)  Bi9g Bang Theory was first proposed by theoretical physicists.

(C)  Big Bang Theory was first proposed by experimental physicists and then deuced by theoretical physicists.

(D)  Philosophers got the Big Bang theory wrong.

(E)  Big Bang theory is not an approximation of the complete truth.

Ans: (B)

Directions (Qs. 23 to 26): Analyze the following passage and provide appropriate answers for these questions :

The base of Objectivism according to Ayan Rand is explicit : “Existence exists-and the act of grasping that statement implies two corollary axioms: that something exists which one perceives and that one exists possessing consciousness, consciousness being the faculty of perceiving that which exists.”

  Existence and consciousness are facts implicit in every perception. They are the base of all knowledge (and the precondition of proof): knowledge presupposes something to know and someone to know it. They are absolutes which cannot be questioned or escaped: every human utterance, including the denial of these axioms, implies their use and acceptance.

   The third axiom at the base of knowledge- an axiom true in Aristotle’s words of “being qua being”-is the Law of Identity. This law defines the essence of existence: to be is to be something, a thing is what it is; and leads to the fundamental principle of all actions, the law of causality. The law of causality states that a thing’s actions are determined not by chance, but by its nature, i.e. by what it is.

  It is important to observe the interrelation of these three axioms. Existence is the first axiom. The universe exists independent of consciousness. Man is able to adapt his background to his own requirements, but “Nature to be commanded, must b e obeyed” (Francis Bacon). There is no mental process that can change the laws of nature or erase facts. The function of consciousness is not to create reality but to apprehend it. “Existence is Identity, Consciousness is Identification.”

23. Which of the following is DEFINETELY CORRECT according to the passage.

(A)  Only what can be perceived exists.

(B)  What exists is perceived.

(C)  All that exists does not have consciousness.

(D)  Consciousness makes perception of being possible.

(E)  Something to be known and someone to know are the condition sime qua non for existence.

Ans: (D)

24. Which of the following is the ESSENCE of ‘The Law of Causality’?

(A)  To be is to be something ‘being qua being’.

(B)  Wishing to become something else denies the nature of that being.

(C)  The law of identity is the same as the law of causality.

(D)  Essence of existence.

(E)  Actions of a being are determined by its nature.

Ans: (E)

25. Which of the following can be best captured as ‘Identity’ and “Identification” ?

(A)  College as identity; perception of cultural events as identification.

(B)  Twitter as identity; perception of Twitter as identification.

(C)  Government as identity; perception of taxation of citizens as identifications.

(D)  Marriage as identity; perception of children as identification.

(E)  MBA as identity; perception of campus placement as identification.

Ans: (B)

26. The author would interpret Francis Bacon’s “Nature to be commanded, must be obeyed” as :

(A)  Reality should not be modified or escaped but faced.

(B)  Man’s existence depends on nature’s whims.

(C)  Essentially and objectively nature is superior to humans.

(D)  Obstacles are better circumvented than confronted.

(E)  Before channeling nature one must first comply with it.

Ans: (E)

© Copyright Entrance India - Engineering and Medical Entrance Exams in India | Website Maintained by Firewall Firm - IT Monteur